Your-Doctor
Multiple Choice Questions (MCQ)


Quiz Categories Click to expand

Category: Critical Care Medicine-Hematologic and Oncologic Disorders--->White Blood Cell Disorders
Page: 1

Question 1# Print Question

A 57-year-old male with history of alcohol abuse is admitted to the intensive care unit after a fall from a ladder. He has sustained a left ankle fracture and several rib fractures. His white blood cell count is 1600/µL.

Initial workup for possible causes of leukopenia includes all of the following EXCEPT:

A. Obtain complete blood count (CBC) with differential
B. Review patient’s medications
C. Obtain screening studies for rheumatologic disorders
D. Review results of CBCs/differential counts from prior hospitalizations and the ambulatory setting


Question 2# Print Question

A 52-year-old female presents with fever of 38.3°C (101°F) and malaise. Her white blood cell count is 710/µL with 70% neutrophils. She was recently diagnosed with an urinary tract infection and is currently taking trimethoprim-sulfamethoxazole.

Which of the following statements is TRUE?

A. Her neutropenia is unlikely to be due to idiosyncratic drug–induced acute neutropenia because her absolute neutrophil count (ANC) is <500/ µL
B. Use of granulocyte colony-stimulating factor (G-CSF) is contraindicated
C. Her prognosis is poor as mortality associated with idiosyncratic drug–induced acute neutropenia is over 50%
D. Vancomycin is not recommended for initial therapy of neutropenic patients with fever


Question 3# Print Question

A 62-year-old male is admitted with fatigue and new-onset shortness of breath. His white blood count is 85,000/µL and bone marrow biopsy shows 35% myeloblasts. He is started on induction chemotherapy for acute myeloid leukemia. During treatment, his creatinine rises and he develops hyperuricemia.

Which of the following statements is FALSE regarding his management?

A. He should receive hydration with intravenous fluids with a target urine output of at least 2 mL/kg/h
B. He should have continuous cardiac monitoring and measurement of electrolytes, creatinine, and uric acid every 4 to 6 hours
C. Rasburicase is contraindicated if he has a glucose-6-phosphate dehydrogenase deficiency
D. His uric acid level will decrease rapidly after starting allopurinol


Question 4# Print Question

A 22-year-old female with a history of bilateral lung transplantation for cystic fibrosis 9 months ago now presents with fever, weight loss, and lymphadenopathy. Lymph node biopsy shows diffuse large Bcell lymphoma.

Treatment includes all of the following EXCEPT:

A. Reduction of immunosuppression
B. Ganciclovir
C. Chemotherapy with cyclophosphamide, doxorubicin, vincristine, and prednisone (CHOP)
D. Rituximab


Question 5# Print Question

All the following are possible long-term complications of treatment of non-Hodgkin lymphoma (NHL) EXCEPT:

A. Breast cancer
B. Stroke
C. Congestive heart failure
D. Adrenal insufficiency




Category: Critical Care Medicine-Hematologic and Oncologic Disorders--->White Blood Cell Disorders
Page: 1 of 2